- Sun Jan 20, 2013 12:00 am
#73942
Complete Question Explanation
Weaken—CE. The correct answer choice is (B)
The premises contain correlations, and the conclusion makes a causal claim:
PC = adequate prenatal care
DR = decrease risk of low birth weight babies
C E
PC DR
The question stem asks you to weaken the argument, and the correct answer falls into one of the five
basic methods for weakening a causal argument.
Answer choice (A): The conclusion specifically states that mothers who had received adequate
prenatal care were less likely to have low birth weight babies than mothers who had received
inadequate prenatal care. Thus, although mothers who received inadequate prenatal care have a
higher likelihood of having low birth weight babies, this likelihood still allows for many babies to
be born of normal weight. In a later chapter we will explore the ways the LSAT uses numbers and
statistics to confuse test takers, but for now, consider this analogy: The Detroit Tigers are more likely
to lose a baseball game than any other team, but even so, they can still win a number of games. In
the same way, the aforementioned mothers may be more likely to have low birth weight babies, but
they can still give birth to babies of normal weight. Hence, answer choice (A) does not attack the
argument.
Answer choice (B): This is the correct answer. The answer choice falls into the category of “Showing
a statistical problem exists with the data used to make the causal statement.” By indicating that
mothers without prenatal care records are routinely classified as mothers receiving inadequate
prenatal care, the answer undermines the relationship in the argument because the data used to make
the conclusion is unreliable.
Answer choice (C): The conclusion is about low birth weight babies, not premature babies. Even if
low birth weight babies were routinely classified as premature, that would not affect the conclusion.
Answer choice (D): Similar to answer choice (A), the likelihoods discussed in the stimulus allow for
this possibility. Hence, this answer cannot hurt the argument.
Answer choice (E): If anything, this answer strengthens the argument since it shows that adequate
prenatal care has a powerful positive effect.
Weaken—CE. The correct answer choice is (B)
The premises contain correlations, and the conclusion makes a causal claim:
PC = adequate prenatal care
DR = decrease risk of low birth weight babies
C E
PC DR
The question stem asks you to weaken the argument, and the correct answer falls into one of the five
basic methods for weakening a causal argument.
Answer choice (A): The conclusion specifically states that mothers who had received adequate
prenatal care were less likely to have low birth weight babies than mothers who had received
inadequate prenatal care. Thus, although mothers who received inadequate prenatal care have a
higher likelihood of having low birth weight babies, this likelihood still allows for many babies to
be born of normal weight. In a later chapter we will explore the ways the LSAT uses numbers and
statistics to confuse test takers, but for now, consider this analogy: The Detroit Tigers are more likely
to lose a baseball game than any other team, but even so, they can still win a number of games. In
the same way, the aforementioned mothers may be more likely to have low birth weight babies, but
they can still give birth to babies of normal weight. Hence, answer choice (A) does not attack the
argument.
Answer choice (B): This is the correct answer. The answer choice falls into the category of “Showing
a statistical problem exists with the data used to make the causal statement.” By indicating that
mothers without prenatal care records are routinely classified as mothers receiving inadequate
prenatal care, the answer undermines the relationship in the argument because the data used to make
the conclusion is unreliable.
Answer choice (C): The conclusion is about low birth weight babies, not premature babies. Even if
low birth weight babies were routinely classified as premature, that would not affect the conclusion.
Answer choice (D): Similar to answer choice (A), the likelihoods discussed in the stimulus allow for
this possibility. Hence, this answer cannot hurt the argument.
Answer choice (E): If anything, this answer strengthens the argument since it shows that adequate
prenatal care has a powerful positive effect.